Finding the HCF











up vote
0
down vote

favorite












Find $(a^{2^m}+1, a^{2^n}+1)$ when a is odd and a,m,n are positive integers and m is not equal to n. I know that the hcf is a multiple of two but I can't prove that it is 2 which is the answer. Plz help.










share|cite|improve this question
























  • Please see math.meta.stackexchange.com/questions/5020
    – Lord Shark the Unknown
    Nov 8 at 5:33










  • math.stackexchange.com/questions/123524/…
    – lab bhattacharjee
    Nov 8 at 5:51










  • Since $a$ is even, both the numbers are odd, so the HCF can't be $2$.
    – B. Goddard
    Nov 8 at 21:07










  • Special case of here where $,2nmid b-c,,$ so the gcd $ =(a+1,2) = 2 $ by $a$ odd.
    – Bill Dubuque
    Nov 12 at 2:55

















up vote
0
down vote

favorite












Find $(a^{2^m}+1, a^{2^n}+1)$ when a is odd and a,m,n are positive integers and m is not equal to n. I know that the hcf is a multiple of two but I can't prove that it is 2 which is the answer. Plz help.










share|cite|improve this question
























  • Please see math.meta.stackexchange.com/questions/5020
    – Lord Shark the Unknown
    Nov 8 at 5:33










  • math.stackexchange.com/questions/123524/…
    – lab bhattacharjee
    Nov 8 at 5:51










  • Since $a$ is even, both the numbers are odd, so the HCF can't be $2$.
    – B. Goddard
    Nov 8 at 21:07










  • Special case of here where $,2nmid b-c,,$ so the gcd $ =(a+1,2) = 2 $ by $a$ odd.
    – Bill Dubuque
    Nov 12 at 2:55















up vote
0
down vote

favorite









up vote
0
down vote

favorite











Find $(a^{2^m}+1, a^{2^n}+1)$ when a is odd and a,m,n are positive integers and m is not equal to n. I know that the hcf is a multiple of two but I can't prove that it is 2 which is the answer. Plz help.










share|cite|improve this question















Find $(a^{2^m}+1, a^{2^n}+1)$ when a is odd and a,m,n are positive integers and m is not equal to n. I know that the hcf is a multiple of two but I can't prove that it is 2 which is the answer. Plz help.







number-theory elementary-number-theory divisibility greatest-common-divisor






share|cite|improve this question















share|cite|improve this question













share|cite|improve this question




share|cite|improve this question








edited Nov 9 at 4:14

























asked Nov 8 at 5:27









Utkarsh Jha

326




326












  • Please see math.meta.stackexchange.com/questions/5020
    – Lord Shark the Unknown
    Nov 8 at 5:33










  • math.stackexchange.com/questions/123524/…
    – lab bhattacharjee
    Nov 8 at 5:51










  • Since $a$ is even, both the numbers are odd, so the HCF can't be $2$.
    – B. Goddard
    Nov 8 at 21:07










  • Special case of here where $,2nmid b-c,,$ so the gcd $ =(a+1,2) = 2 $ by $a$ odd.
    – Bill Dubuque
    Nov 12 at 2:55




















  • Please see math.meta.stackexchange.com/questions/5020
    – Lord Shark the Unknown
    Nov 8 at 5:33










  • math.stackexchange.com/questions/123524/…
    – lab bhattacharjee
    Nov 8 at 5:51










  • Since $a$ is even, both the numbers are odd, so the HCF can't be $2$.
    – B. Goddard
    Nov 8 at 21:07










  • Special case of here where $,2nmid b-c,,$ so the gcd $ =(a+1,2) = 2 $ by $a$ odd.
    – Bill Dubuque
    Nov 12 at 2:55


















Please see math.meta.stackexchange.com/questions/5020
– Lord Shark the Unknown
Nov 8 at 5:33




Please see math.meta.stackexchange.com/questions/5020
– Lord Shark the Unknown
Nov 8 at 5:33












math.stackexchange.com/questions/123524/…
– lab bhattacharjee
Nov 8 at 5:51




math.stackexchange.com/questions/123524/…
– lab bhattacharjee
Nov 8 at 5:51












Since $a$ is even, both the numbers are odd, so the HCF can't be $2$.
– B. Goddard
Nov 8 at 21:07




Since $a$ is even, both the numbers are odd, so the HCF can't be $2$.
– B. Goddard
Nov 8 at 21:07












Special case of here where $,2nmid b-c,,$ so the gcd $ =(a+1,2) = 2 $ by $a$ odd.
– Bill Dubuque
Nov 12 at 2:55






Special case of here where $,2nmid b-c,,$ so the gcd $ =(a+1,2) = 2 $ by $a$ odd.
– Bill Dubuque
Nov 12 at 2:55












1 Answer
1






active

oldest

votes

















up vote
0
down vote













a is odd, let $a=2k+1$ then:



$a^{2^m}+1=(2k+1)^{2^m}+1= 2t+2=2(t+1)$



Where due to binomial theorem t is a sum with factors $(2k)^i$ and $2^m$. Similarly we have:



$a^{2^n}+1=(2k+1)^{2^n}+1= 2s+2=2(s+1)$



Where s is the sum of terms like $(2k)^i C^{2^n}_i$. Therefore we may write:



$(a^{2m}+1, a^{2^n}+1)=2$






share|cite|improve this answer





















    Your Answer





    StackExchange.ifUsing("editor", function () {
    return StackExchange.using("mathjaxEditing", function () {
    StackExchange.MarkdownEditor.creationCallbacks.add(function (editor, postfix) {
    StackExchange.mathjaxEditing.prepareWmdForMathJax(editor, postfix, [["$", "$"], ["\\(","\\)"]]);
    });
    });
    }, "mathjax-editing");

    StackExchange.ready(function() {
    var channelOptions = {
    tags: "".split(" "),
    id: "69"
    };
    initTagRenderer("".split(" "), "".split(" "), channelOptions);

    StackExchange.using("externalEditor", function() {
    // Have to fire editor after snippets, if snippets enabled
    if (StackExchange.settings.snippets.snippetsEnabled) {
    StackExchange.using("snippets", function() {
    createEditor();
    });
    }
    else {
    createEditor();
    }
    });

    function createEditor() {
    StackExchange.prepareEditor({
    heartbeatType: 'answer',
    convertImagesToLinks: true,
    noModals: true,
    showLowRepImageUploadWarning: true,
    reputationToPostImages: 10,
    bindNavPrevention: true,
    postfix: "",
    imageUploader: {
    brandingHtml: "Powered by u003ca class="icon-imgur-white" href="https://imgur.com/"u003eu003c/au003e",
    contentPolicyHtml: "User contributions licensed under u003ca href="https://creativecommons.org/licenses/by-sa/3.0/"u003ecc by-sa 3.0 with attribution requiredu003c/au003e u003ca href="https://stackoverflow.com/legal/content-policy"u003e(content policy)u003c/au003e",
    allowUrls: true
    },
    noCode: true, onDemand: true,
    discardSelector: ".discard-answer"
    ,immediatelyShowMarkdownHelp:true
    });


    }
    });














    draft saved

    draft discarded


















    StackExchange.ready(
    function () {
    StackExchange.openid.initPostLogin('.new-post-login', 'https%3a%2f%2fmath.stackexchange.com%2fquestions%2f2989562%2ffinding-the-hcf%23new-answer', 'question_page');
    }
    );

    Post as a guest















    Required, but never shown

























    1 Answer
    1






    active

    oldest

    votes








    1 Answer
    1






    active

    oldest

    votes









    active

    oldest

    votes






    active

    oldest

    votes








    up vote
    0
    down vote













    a is odd, let $a=2k+1$ then:



    $a^{2^m}+1=(2k+1)^{2^m}+1= 2t+2=2(t+1)$



    Where due to binomial theorem t is a sum with factors $(2k)^i$ and $2^m$. Similarly we have:



    $a^{2^n}+1=(2k+1)^{2^n}+1= 2s+2=2(s+1)$



    Where s is the sum of terms like $(2k)^i C^{2^n}_i$. Therefore we may write:



    $(a^{2m}+1, a^{2^n}+1)=2$






    share|cite|improve this answer

























      up vote
      0
      down vote













      a is odd, let $a=2k+1$ then:



      $a^{2^m}+1=(2k+1)^{2^m}+1= 2t+2=2(t+1)$



      Where due to binomial theorem t is a sum with factors $(2k)^i$ and $2^m$. Similarly we have:



      $a^{2^n}+1=(2k+1)^{2^n}+1= 2s+2=2(s+1)$



      Where s is the sum of terms like $(2k)^i C^{2^n}_i$. Therefore we may write:



      $(a^{2m}+1, a^{2^n}+1)=2$






      share|cite|improve this answer























        up vote
        0
        down vote










        up vote
        0
        down vote









        a is odd, let $a=2k+1$ then:



        $a^{2^m}+1=(2k+1)^{2^m}+1= 2t+2=2(t+1)$



        Where due to binomial theorem t is a sum with factors $(2k)^i$ and $2^m$. Similarly we have:



        $a^{2^n}+1=(2k+1)^{2^n}+1= 2s+2=2(s+1)$



        Where s is the sum of terms like $(2k)^i C^{2^n}_i$. Therefore we may write:



        $(a^{2m}+1, a^{2^n}+1)=2$






        share|cite|improve this answer












        a is odd, let $a=2k+1$ then:



        $a^{2^m}+1=(2k+1)^{2^m}+1= 2t+2=2(t+1)$



        Where due to binomial theorem t is a sum with factors $(2k)^i$ and $2^m$. Similarly we have:



        $a^{2^n}+1=(2k+1)^{2^n}+1= 2s+2=2(s+1)$



        Where s is the sum of terms like $(2k)^i C^{2^n}_i$. Therefore we may write:



        $(a^{2m}+1, a^{2^n}+1)=2$







        share|cite|improve this answer












        share|cite|improve this answer



        share|cite|improve this answer










        answered Nov 17 at 17:16









        sirous

        1,5391513




        1,5391513






























            draft saved

            draft discarded




















































            Thanks for contributing an answer to Mathematics Stack Exchange!


            • Please be sure to answer the question. Provide details and share your research!

            But avoid



            • Asking for help, clarification, or responding to other answers.

            • Making statements based on opinion; back them up with references or personal experience.


            Use MathJax to format equations. MathJax reference.


            To learn more, see our tips on writing great answers.





            Some of your past answers have not been well-received, and you're in danger of being blocked from answering.


            Please pay close attention to the following guidance:


            • Please be sure to answer the question. Provide details and share your research!

            But avoid



            • Asking for help, clarification, or responding to other answers.

            • Making statements based on opinion; back them up with references or personal experience.


            To learn more, see our tips on writing great answers.




            draft saved


            draft discarded














            StackExchange.ready(
            function () {
            StackExchange.openid.initPostLogin('.new-post-login', 'https%3a%2f%2fmath.stackexchange.com%2fquestions%2f2989562%2ffinding-the-hcf%23new-answer', 'question_page');
            }
            );

            Post as a guest















            Required, but never shown





















































            Required, but never shown














            Required, but never shown












            Required, but never shown







            Required, but never shown

































            Required, but never shown














            Required, but never shown












            Required, but never shown







            Required, but never shown







            Popular posts from this blog

            QoS: MAC-Priority for clients behind a repeater

            Ивакино (Тотемский район)

            Can't locate Autom4te/ChannelDefs.pm in @INC (when it definitely is there)